Регистрирайте сеРегистрирайте се

Екстремални Задачи и Решения

Иди на страница Предишна  1, 2, 3, 4, 5, 6, 7, 8  Следваща
 
   Форум за математика Форуми -> Олимпиади и състезания за 9-12 клас
Предишната тема :: Следващата тема  
Автор Съобщение
Titu_Andrescu
Напреднал


Регистриран на: 28 Oct 2006
Мнения: 370

Репутация: 68.9
гласове: 29

МнениеПуснато на: Mon Jan 14, 2008 12:51 pm    Заглавие:

Задача 75*. Нека [tex]a,b,c[/tex] са страни на триъгълник с лице [tex]S[/tex] и нека [tex]u,v,w[/tex] са положителни реални числа. Да се докаже, че
[tex]\frac{ua^4}{v+w}+\frac{vb^4}{w+u}+\frac{wc^4}{u+v}\ge 8S^2[/tex].
Кога се достига равенство?
(Джордж Тсинтсифас, Гърция)
Върнете се в началото
Вижте профила на потребителя Изпратете лично съобщение
Реклама







Пуснато на:     Заглавие: Реклама

Върнете се в началото
Мирослав Стоенчев
Напреднал


Регистриран на: 21 Aug 2007
Мнения: 279

Репутация: 72
гласове: 45

МнениеПуснато на: Mon Jan 14, 2008 1:46 pm    Заглавие:

Задача 75. Д-во: [tex]\frac{wa^{4}}{u+v}+\frac{ub^{4}}{v+w}+\frac{vc^{4}}{w+u}\ge 8S^{2}\Leftrightarrow \frac{wa^{4}}{u+v}+\frac{ub^{4}}{v+w}+\frac{vc^{4}}{w+u}\ge a^{2}b^{2}+b^{2}c^{2}+c^{2}a^{2}-\frac{a^{4}+b^{4}+c^{4}}{2}\Leftrightarrow (u+v+w)\left\(\frac{a^{4}}{u+v}+\frac{b^{4}}{v+w}+\frac{c^{4}}{w+u}\right\)\ge \frac{(a^{2}+b^{2}+c^{2})^{2}}{2}\Leftrightarrow[/tex]

[tex]2(u+v+w)\left\(\frac{a^{4}}{u+v}+\frac{b^{4}}{v+w}+\frac{c^{4}}{w+u}\right\)\ge (a^{2}+b^{2}+c^{2})^{2}\Leftrightarrow \left\((u+v)+(v+w)+(w+u)\right\)\left\(\frac{a^{4}}{u+v}+\frac{b^{4}}{v+w}+\frac{c^{4}}{w+u}\right\)\ge (a^{2}+b^{2}+c^{2})^{2}.[/tex]

От неравеснството на Коши-Буняковски-Шварц получаваме

[tex]\left\((u+v)+(v+w)+(w+u)\right\)\left\(\frac{a^{4}}{u+v}+\frac{b^{4}}{v+w}+\frac{c^{4}}{w+u}\right\)\ge \left\(\sqrt{(u+v)\frac{a^{4}}{u+v}}+\sqrt{(v+w)\frac{b^{4}}{v+w}}+\sqrt{(w+u)\frac{c^{4}}{w+u}}\right\)^{2}=(a^{2}+b^{2}+c^{2})^{2}.[/tex]

Равенство се достига при [tex]\frac{a^{2}}{u+v}=\frac{b^{2}}{v+w}=\frac{c^{2}}{w+u}.[/tex] Неравенството е доказано.
Върнете се в началото
Вижте профила на потребителя Изпратете лично съобщение
Titu_Andrescu
Напреднал


Регистриран на: 28 Oct 2006
Мнения: 370

Репутация: 68.9
гласове: 29

МнениеПуснато на: Mon Jan 14, 2008 3:52 pm    Заглавие:

Забележете, че ако положим [tex]u=h_a^4,v=h_b^4,w=h_c^4[/tex] в задача 75 ще докажем задача 71 б). Следователно 75 е по-силно неравенство от 71 б).
Върнете се в началото
Вижте профила на потребителя Изпратете лично съобщение
Titu_Andrescu
Напреднал


Регистриран на: 28 Oct 2006
Мнения: 370

Репутация: 68.9
гласове: 29

МнениеПуснато на: Mon Jan 14, 2008 5:04 pm    Заглавие:

Задача 76. Да се докаже, че за всички положителни реални [tex]x,y,z[/tex]:
a)[tex]\frac{x}{\sqrt{x+y}}+\frac{y}{\sqrt{y+z}}+\frac{z}{\sqrt{z+x}}> \frac{\sqrt{x}+\sqrt{y}+\sqrt{z}}{2}[/tex] .
b)*[tex]\frac{x}{\sqrt{x+y}}+\frac{y}{\sqrt{y+z}}+\frac{z}{\sqrt{z+x}}\ge \frac{\sqrt{x}+\sqrt{y}+\sqrt{z}}{\sqrt{2}}[/tex] (Лолтер Джонс, Инсбург, Австрия)

Задача 77*. В списание Math. Gazette 68 (1984 г.), P. Stanbury отбелязва интересното приближение, че [tex]\frac{\pi^9}{e^8}\approx 9.999838813\approx 10[/tex].
Съществуват ли естествени числа [tex]l[/tex] и [tex]m[/tex], за които числото [tex] \frac{\pi^l}{e^m}[/tex] е по-близо до цяло число от показания по-горе пример?
(Murray S. Klamkin)

Задача 78. Нека [tex]a,b,c[/tex] и [tex]m_a,m_b,m_c[/tex] са съответно страните и медианите на даден триъгълник. Да се намери множеството от всички реални числа [tex]t[/tex] и за всяко такова число [tex]t[/tex]
да се намери най-голямата положителна константа [tex]\lambda_t[/tex], за която е изпълнено
[tex]\frac{m_am_bm_c}{abc}\ge\lambda_t\frac{m_a^t+m_b^t+m_c^t}{a+b+c}[/tex].
(Лолтер Джонс, Инсбург, Австрия)

Задача 79. Да се докаже, че за три положителни реални числа [tex]a,b,c[/tex] с произведение [tex]1[/tex] и произволно реално число [tex]k\ge 0[/tex] e в сила неравенството
[tex]\frac{1}{1+a+b^k}+\frac{1}{1+b+c^k}+\frac{1}{1+c+a^k}\le 1[/tex].
(Vasile Cirtoaje)


Последната промяна е направена от Titu_Andrescu на Sun Sep 13, 2009 3:44 pm; мнението е било променяно общо 6 пъти
Върнете се в началото
Вижте профила на потребителя Изпратете лично съобщение
Мирослав Стоенчев
Напреднал


Регистриран на: 21 Aug 2007
Мнения: 279

Репутация: 72
гласове: 45

МнениеПуснато на: Mon Jan 14, 2008 5:51 pm    Заглавие:

Задача 76 a). Полагаме [tex]x=a^{2},y=b^{2},z=c^{2},[/tex] [tex]a,b,c\in R^{+}\cup \left\{0\right\}.[/tex] Трябва да докажем, че [tex]\frac{a^{2}}{\sqrt{a^{2}+b^{2}}}+\frac{b^{2}}{\sqrt{b^{2}+c^{2}}}+\frac{c^{2}}{\sqrt{c^{2}+a^{2}}}> \frac{a+b+c}{2}.[/tex]

Чрез прилагане неравенството на Коши-Буняковски-Шварц получаваме

[tex]\left\(\sqrt{a^{2}+b^{2}}+\sqrt{b^{2}+c^{2}}+\sqrt{c^{2}+a^{2}}\right\)\left\(\frac{a^{2}}{\sqrt{a^{2}+b^{2}}}+\frac{b^{2}}{\sqrt{b^{2}+c^{2}}}+\frac{c^{2}}{\sqrt{c^{2}+a^{2}}}\right\)\ge \left\(\sqrt{\sqrt{a^{2}+b^{2}}\frac{a^{2}}{\sqrt{a^{2}+b^{2}}}}+\sqrt{\sqrt{b^{2}+c^{2}}\frac{b^{2}}{\sqrt{b^{2}+c^{2}}}}+\sqrt{\sqrt{b^{2}+c^{2}}\frac{c^{2}}{\sqrt{c^{2}+a^{2}}}}\right\)^{2}=(a+b+c)^{2}[/tex]

[tex]\Rightarrow \frac{a^{2}}{\sqrt{a^{2}+b^{2}}}+\frac{b^{2}}{\sqrt{b^{2}+c^{2}}}+\frac{c^{2}}{\sqrt{c^{2}+a^{2}}}\ge \frac{(a+b+c)^{2}}{\sqrt{a^{2}+b^{2}}+\sqrt{b^{2}+c^{2}}+\sqrt{c^{2}+a^{2}}}.[/tex]

Ще докажем, че [tex]\frac{(a+b+c)^{2}}{\sqrt{a^{2}+b^{2}}+\sqrt{b^{2}+c^{2}}+\sqrt{c^{2}+a^{2}}}> \frac{a+b+c}{2}\Leftrightarrow 2(a+b+c)> \sqrt{a^{2}+b^{2}}+\sqrt{b^{2}+c^{2}}+\sqrt{c^{2}+a^{2}}\Leftrightarrow \left\(a+b-\sqrt{a^{2}+b^{2}}\right\)+\left\(b+c-\sqrt{b^{2}+c^{2}}\right\)+\left\(c+a-\sqrt{c^{2}+a^{2}}\right\)> 0.[/tex]

Последното неравенство е в сила понеже при положителни [tex]a,b\Rightarrow a+b> \sqrt{a^{2}+b^{2}}.[/tex] Неравенството е доказано.
Върнете се в началото
Вижте профила на потребителя Изпратете лично съобщение
kamen05
Начинаещ


Регистриран на: 22 Oct 2007
Мнения: 34
Местожителство: Sofia
Репутация: 25.8Репутация: 25.8Репутация: 25.8
гласове: 18

МнениеПуснато на: Tue Jan 15, 2008 9:48 pm    Заглавие: Задача 77*

Titu_Andrescu написа:
Задача 76. Да се докаже, че за всички положителни реални [tex]x,y,z[/tex]:
a)[tex]\frac{x}{\sqrt{x+y}}+\frac{y}{\sqrt{y+z}}+\frac{z}{\sqrt{z+x}}> \frac{\sqrt{x}+\sqrt{y}+\sqrt{z}}{2}[/tex] .
b)*[tex]\frac{x}{\sqrt{x+y}}+\frac{y}{\sqrt{y+z}}+\frac{z}{\sqrt{z+x}}\ge \frac{\sqrt{x}+\sqrt{y}+\sqrt{z}}{\sqrt{2}}[/tex] (Лолтер Джонс, Инсбург, Австрия)

Задача 77*. В списание Math. Gazette 68 (1984 г.), P. Stanbury отбелязва интересното приближение, че [tex]\frac{\pi^9}{e^8}\approx 9.999838813\approx 10[/tex].
Съществуват ли естествени числа [tex]l[/tex] и [tex]m[/tex], за които числото [tex] \frac{\pi^l}{e^m}[/tex] е по-близо до цяло число от показания по-горе пример?
(Murray S. Klamkin)

Задача 78. Нека [tex]a,b,c[/tex] и [tex]m_a,m_b,m_c[/tex] са съответно страните и медианите на даден триъгълник. Да се намери множеството от всички реални числа [tex]t[/tex] и за всяко такова число [tex]t[/tex] да се намери най-голямата положителна константа [tex]\lambda_t[/tex], за която е изпълнено
[tex]\frac{m_am_bm_c}{abc}\ge\lambda_t\frac{m_a^t+m_b^t+m_c^t}{a+b+c}[/tex].
(Лолтер Джонс, Инсбург, Австрия)

Задача 79. Да се докаже, че за три положителни реални числа [tex]a,b,c[/tex] с произведение [tex]1[/tex] и произволно реално число [tex]k\ge 0[/tex] e в сила неравенството
[tex]\frac{1}{1+a+b^k}+\frac{1}{1+b+c^k}+\frac{1}{1+c+a^k}\le 1[/tex].
(Vasile Cirtoaje)

Да приемем, че [tex] ln(\pi ) \in R/Q [/tex] Тогава { {[tex]k.ln(\pi )[/tex]} }е гъсто множество за [tex]k\in N [/tex], където с [tex]\left\{x\right\}[/tex] сме отбелязали дробната част на x т.е.
[tex]\left\{x\right\}=x-[x][/tex].
Доказателството на този факт може да се намери на много места, който иска- мога да му го изпратя, но предвид, че разделът е "Олимпиади", не мисля че е нужно. Very Happy
Нека t положително число по-малко от [tex]ln( 1+10-\frac{\pi^9}{e^8}) [/tex]. Нека сега K е такова естествено число (а то съществува, поради гъстотата), че [tex] 0\le K ln(\pi) - [K ln(\pi)]\le t [/tex] или
[tex] e^0\le e^{K ln(\pi) - [K ln(\pi)]}\le e^t<1+10-\frac{\pi^9}{e^8}[/tex] или [tex]1\le \frac{\pi^K}{e^{[K.ln(\pi )]}}[/tex][tex]<1+10-\frac{\pi^9}{e^8}[/tex] Т.е. [tex]l=K, m=[K.ln(\pi )] [/tex] е по-добро приближение до 1 отколкото това в условието - до 10.
И дотук щяхме да сме свършили, ако бяхме доказали "незначителния" факт [tex] ln(\pi ) \in R/Q [/tex]. Честно казано не само не се сещам как да го докажа със средствата на "средношколската" математика, ами май въобще съм забравил как се доказваше. Но един внимателен прочит на условието на задачата ме наведе на мисълта, че мога да изхитрувам съвсем законно например така: Да допуснем, че [tex] ln(\pi ) \in Q [/tex].
Тогава има естествени числа [tex]p[/tex] и [tex] q [/tex], такива че [tex]\pi ^q=e^p[/tex] и тогава [tex] l=q, m=p [/tex] е по-добро приближение до 1 отколкото това в условието - до 10. Всъщност с това последно разсъждение доказахме, че отговорът на задачката е твърдо "ДА"! Very Happy
Върнете се в началото
Вижте профила на потребителя Изпратете лично съобщение
Titu_Andrescu
Напреднал


Регистриран на: 28 Oct 2006
Мнения: 370

Репутация: 68.9
гласове: 29

МнениеПуснато на: Wed Jan 16, 2008 11:17 am    Заглавие:

kamen05, винаги ме и било безкраино приятно да чета разсъжденията ти.
Върнете се в началото
Вижте профила на потребителя Изпратете лично съобщение
Nona
Напреднал


Регистриран на: 12 Sep 2006
Мнения: 477

Репутация: 234.7
гласове: 163

МнениеПуснато на: Wed Jan 16, 2008 5:25 pm    Заглавие:

Titu_Andrescu написа:
Задача 71. Нека [tex]h_a,h_b,h_c[/tex] са височините на даден триъгълник с лице [tex]S[/tex]. Да се докаже, че

a) [tex]h_a^4+h_b^4+h_c^4\ge 9S^2[/tex], ако триъгълникът е остроъгълен.
(Мирослав Стоенчев)



[tex]h_a=\frac{2S}{a}\Rightarrow h_a^4+h_b^4+h_c^4\ge 9S^2\Leftrightarrow \frac{16S^4}{a^4}+\frac{16S^4}{b^4}+\frac{16S^4}{c^4}\ge 9S^2 \\ 16S^2\left ( \frac{1}{a^4}+\frac{1}{b^4}+\frac{1}{c^4}\right ) \ge 9[/tex]

От Хероновата формула получаваме:

[tex]S=\sqrt{p(p-a)(p-b)(p-c)}\Leftrightarrow 16S^2=-\sum_{cyc}{a^4}+2\sum_{cyc}{a^2b^2}.[/tex]

Нека [tex]l=a^2, \ m=b^2, \ n=c^2.[/tex]

[tex]\Rightarrow 16S^2=-\sum_{cyc}{l^2}+2\sum_{cyc}{l^2m^2}=\sum_{cyc}{(m+n-l)(l+n-m)}[/tex]

Трябва да докажем, че:

[tex]\sum_{cyc}{(m+n-l)(l+n-m)}\sum_{cyc}{\frac1{l^2}}\ge 9[/tex]

Полагаме [tex]x=m+n-l, \ y=l+m-n, \ z=l+n-m.[/tex] Забележете, че [tex]x=b^2+c^2-a^2=2bc\cos{A}[/tex].
Понеже триъгълникът е остроъгълен, [tex]x, \ y, \ >0.[/tex] Неравенството е еквивалентно на

[tex]\sum_{cyc}{xy}\sum_{cyc}{\frac1{(x+y)^2}\ge \frac 94,[/tex]

което е известно като ВЕЛИКОТО ИРАНСКО НЕРАВЕНСТВО (Задача 4*).
Върнете се в началото
Вижте профила на потребителя Изпратете лично съобщение
Мирослав Стоенчев
Напреднал


Регистриран на: 21 Aug 2007
Мнения: 279

Репутация: 72
гласове: 45

МнениеПуснато на: Fri Jan 18, 2008 7:26 pm    Заглавие:

Задача 66. Д-во: Неравенството [tex]d^{2}+e^{2}+f^{2}\le \frac{p^{2}}{3}[/tex] е хомогенно, следователно б.о.о. ще считаме, че [tex]a+b+c=2p=1.[/tex]

Нека [tex]AD,\ BE,\ CF [/tex] са вътрешните ъглополовящи на [tex]\triangle ABC,\ D\in BC,\ E\in CA,\ F\in AB.[/tex] Полагаме [tex]d=|EF|,\ e=|FD|,\ f=|DE|.[/tex] По косинусова теорема за триъгълниците [tex]\triangle AFE,\ \triangle BDF,\ \triangle CED\Rightarrow [/tex]

[tex]d^{2}=\left\(\frac{bc}{c+a}\right\)^{2}+\left\(\frac{bc}{a+b}\right\)^{2}-\frac{bc(b^{2}+c^{2}-a^{2})}{(c+a)(a+b)}[/tex]

[tex]e^{2}=\left\(\frac{ac}{a+b}\right\)^{2}+\left\(\frac{ac}{b+c}\right\)^{2}-\frac{ac(c^{2}+a^{2}-b^{2})}{(a+b)(b+c)}[/tex]

[tex]f^{2}=\left\(\frac{ab}{b+c}\right\)^{2}+\left\(\frac{ab}{c+a}\right\)^{2}-\frac{ab(a^{2}+b^{2}-c^{2})}{(b+c)(c+a)}[/tex]


Трябва да докажем, че [tex]d^{2}+e^{2}+f^{2}\le \frac{1}{12}\Leftrightarrow \sum_{cyclic}\left\[\left\(\frac{bc}{c+a}\right\)^{2}+\left\(\frac{bc}{a+b}\right\)^{2}-\frac{bc(b^{2}+c^{2}-a^{2})}{(c+a)(a+b)}\right\]\le\frac{1}{12}. [/tex]

Полагаме [tex]G(x,y,z)=\sum_{cyclic}\left\[\left\(\frac{yz}{z+x}\right\)^{2}+\left\(\frac{yz}{x+y}\right\)^{2}-\frac{yz(y^{2}+z^{2}-x^{2})}{(z+x)(x+y)}\right\], x+y+z=1,\ \frac{1}{3}\le \max\left\{x,y,z\right\}<\frac{1}{2}.[/tex]

[tex]*[/tex]

Нека сега [tex]x+2y=1,\ \frac{1}{4}<y<\frac{1}{2}.[/tex] Пресмятаме [tex]G(x,y,y)=\frac{x^{2}(6y^{2}+(x+y)^{2}-2x(x+y)}{2(x+y)^{2}}\le \frac{1}{12}\Leftrightarrow (3y-1)^{2}(8y^{2}+8y-7)\le 0, [/tex] което е изпълнено. Доказахме, че [tex]G(x,y,y)\le \frac{1}{12}.[/tex]

[tex]**[/tex]

Нека сега [tex]x+y+z=1,\ \min\left\{x,y,z\right\}=x.[/tex] Пресмятаме разликата [tex]G(x,y,z)-G(x,\frac{y+z}{2},\frac{y+z}{2})=[/tex]

[tex]=-(y-z)^{2}\left\{\frac{1}{4}-\left\(\frac{yz}{(z+x)(x+y)}-\frac{1}{2}\right\)^{2}+\frac{x(x+y+z)\left\[x^{2}(y+z)+(x+y+z)^{2}(2x+y+z)\right\]}{(x+y)(y+z)(z+x)(2x+y+z)^{2}}-\frac{x^{2}}{2(y+z)^{2}}-\frac{x^{2}(x+y+z)\left\[(y+z)(x+y+z)(2x+y+z)+2x\left\((x+y+z)^{2}-yz\right\)\right\]}{(z+x)^{2}(x+y)^{2}(2x+y+z)^{2}}\right\}\le 0,[/tex]

понеже [tex]\ \frac{1}{4}-\left\(\frac{yz}{(z+x)(x+y)}-\frac{1}{2}\right\)^{2}+\frac{x(x+y+z)\left\[x^{2}(y+z)+(x+y+z)^{2}(2x+y+z)\right\]}{(x+y)(y+z)(z+x)(2x+y+z)^{2}}-\frac{x^{2}}{2(y+z)^{2}}-\frac{x^{2}(x+y+z)\left\[(y+z)(x+y+z)(2x+y+z)+2x\left\((x+y+z)^{2}-yz\right\)\right\]}{(z+x)^{2}(x+y)^{2}(2x+y+z)^{2}}\ge 0,[/tex]

при положителни [tex]x,y,z[/tex] за които [tex]\ \min\left\{x,y,z\right\}=x.[/tex]

С това доказахме, че [tex]G(x,y,z)-G(x,\frac{y+z}{2},\frac{y+z}{2})\le 0\Leftrightarrow G(x,y,z)\le G(x,\frac{y+z}{2},\frac{y+z}{2})\le \max_{\frac{1}{4}<y<\frac{1}{2}}G(x,y,y)=\frac{1}{12}\Rightarrow G(x,y,z)\le \frac{1}{12}.[/tex] Неравенството е доказано.


Последната промяна е направена от Мирослав Стоенчев на Sat Jan 19, 2008 4:45 pm; мнението е било променяно общо 1 път
Върнете се в началото
Вижте профила на потребителя Изпратете лично съобщение
kamen05
Начинаещ


Регистриран на: 22 Oct 2007
Мнения: 34
Местожителство: Sofia
Репутация: 25.8Репутация: 25.8Репутация: 25.8
гласове: 18

МнениеПуснато на: Fri Jan 18, 2008 8:03 pm    Заглавие: Решение на 76,б*

Titu_Andrescu написа:
Задача 76. Да се докаже, че за всички положителни реални [tex]x,y,z[/tex]:
a)[tex]\frac{x}{\sqrt{x+y}}+\frac{y}{\sqrt{y+z}}+\frac{z}{\sqrt{z+x}}> \frac{\sqrt{x}+\sqrt{y}+\sqrt{z}}{2}[/tex] .
b)*[tex]\frac{x}{\sqrt{x+y}}+\frac{y}{\sqrt{y+z}}+\frac{z}{\sqrt{z+x}}\ge \frac{\sqrt{x}+\sqrt{y}+\sqrt{z}}{\sqrt{2}}[/tex] (Лолтер Джонс, Инсбург, Австрия)

Задача 77*. В списание Math. Gazette 68 (1984 г.), P. Stanbury отбелязва интересното приближение, че [tex]\frac{\pi^9}{e^8}\approx 9.999838813\approx 10[/tex].
Съществуват ли естествени числа [tex]l[/tex] и [tex]m[/tex], за които числото [tex] \frac{\pi^l}{e^m}[/tex] е по-близо до цяло число от показания по-горе пример?
(Murray S. Klamkin)

Задача 78. Нека [tex]a,b,c[/tex] и [tex]m_a,m_b,m_c[/tex] са съответно страните и медианите на даден триъгълник. Да се намери множеството от всички реални числа [tex]t[/tex] и за всяко такова число [tex]t[/tex] да се намери най-голямата положителна константа [tex]\lambda_t[/tex], за която е изпълнено
[tex]\frac{m_am_bm_c}{abc}\ge\lambda_t\frac{m_a^t+m_b^t+m_c^t}{a+b+c}[/tex].
(Лолтер Джонс, Инсбург, Австрия)

Задача 79. Да се докаже, че за три положителни реални числа [tex]a,b,c[/tex] с произведение [tex]1[/tex] и произволно реално число [tex]k\ge 0[/tex] e в сила неравенството
[tex]\frac{1}{1+a+b^k}+\frac{1}{1+b+c^k}+\frac{1}{1+c+a^k}\le 1[/tex].
(Vasile Cirtoaje)


Първо ще фомулирам без доказателство една лема. тя е съвсем естествена, има обобщение, но го правя за прегледност.
Лема; Ако [tex]a_1\ge a_2\ge a_3, b_1\ge b_2\ge b_3 \Rightarrow a_1.b_1+a_3.b_2+a_2.b_3\ge a_1.b_3+a_3.b_2+a_2.b_1\ge a_1.b_3+a_2.b_2+a_3.b_1 [/tex]
Всъщност от тази лема ни интересува само това, че първата сума е по-голяма от третата. Междинната съм я сложил, за да се види доказателството на лемата.
Сега да пристъпим към задачата. Първо без ограничение ще считаме, че [tex]x=X^2, y=Y^2, z=Z^2[/tex]. Повдигаме двете страни на втора степен и се получава, че трябва да докажем:
[tex]\sum_{cyc}^{}\frac{X^4}{X^2+Y^2}+2\sum_{cyc}^{}\frac{X^2.Y^2}{\sqrt{X^2+Y^2}.\sqrt{Y^2+Z^2}} \ge \frac{(X+Y+Z)^2}{2} [/tex]
Да разгледаме двете функции [tex] F(X,Y)=\frac{X^2.Y^2}{\sqrt{X^2+Y^2}}, G(X,Y)=\frac{1}{\sqrt{X^2+Y^2}} [/tex]. Втората е намаляваща и по двете си компоненти, а първата - растяща по двете си компоненти, защото [tex] F^{`}_X=\frac{X^3.Y^2+2X.Y^4}{(X^2+Y^2)^{\frac{3}{2}}} [/tex]. Сега предвид лемата (ще го разпиша подробно за да се види):
[tex]F(X,Y).G(Y,Z)+F(Y,Z).G(Z,X)+F(Z,X).G(X,Y) \ge F(X,Y).G(X,Y)+F(Y,Z).G(Y,Z)+F(Z,X).G(Z,X) [/tex]
или
[tex] \frac{X^2.Y^2}{\sqrt{X^2+Y^2}}.\frac{1}{.\sqrt{Y^2+Z^2}}+\frac{Y^2.Z^2}{\sqrt{Y^2+Z^2}}.\frac{1}{.\sqrt{Z^2+X^2}}+\frac{Z^2.X^2}{\sqrt{Z^2+X^2}}.\frac{1}{.\sqrt{X^2+Y^2}}\ge \frac{X^2.Y^2}{\sqrt{X^2+Y^2}}.\frac{1}{.\sqrt{X^2+Y^2}}+\frac{Y^2.Z^2}{\sqrt{Y^2+Z^2}}.\frac{1}{.\sqrt{Y^2+Z^2}}+\frac{Z^2.X^2}{\sqrt{Z^2+X^2}}.\frac{1}{.\sqrt{Z^2+X^2}}=\sum_{cyc}^{} \frac{X^2.Y^2}{X^2+Y^2}[/tex]
И това ни дава едно симетрично неравенство, доказателството на което решава задачата:
[tex] \sum_{cyc}^{}\frac{X^4+2X^2.Y^2}{X^2+Y^2}\ge \frac{(X+Y+Z)^2}{2}\Leftarrow 2.\sum_{}^{ } X^2+2.\sum_{}^{ } \frac{X^2.Y^2}{X^2+Y^2} \ge \sum_{}^{ } X^2+2.\sum_{}^{ } X.Y \Leftarrow \sum_{}^{ }X^2+\sum_{}^{ } \frac{2.X^2.Y^2}{X^2+Y^2}\ge 2.\sum_{}^{ } X.Y \Leftarrow \sum_{}^{ } \frac{X^2+Y^2}{2}+\frac{2.X^2Y^2}{X^2+Y^2}-2.X.Y\ge 0\Leftarrow \sum_{}^{ } \frac{(X^2+Y^2)^2+4.X^2.Y^2-4.X.Y.(X^2+Y^2)}{2.(X^2+Y^2)}\ge 0\Leftarrow \sum_{}^{ }\frac{(X-Y)^4}{2.(X^2+Y^2)}\ge 0 [/tex] Готово!
Върнете се в началото
Вижте профила на потребителя Изпратете лично съобщение
kamen05
Начинаещ


Регистриран на: 22 Oct 2007
Мнения: 34
Местожителство: Sofia
Репутация: 25.8Репутация: 25.8Репутация: 25.8
гласове: 18

МнениеПуснато на: Fri Jan 18, 2008 8:22 pm    Заглавие: Относно зад.66

Мирослав Стоенчев, не съм и предполагал, че задача 66 може да се "изкомпенсира"! Не знам защо не мога да ти дам репутация за това решение (дава ми някакво съобщение, което не разбирам) и затова пиша директно във форума!
Решението ти е наистина перфектно за тази изключително трудна задача!
Модераторите да ме извинят, че им изземвам коментарните функции, но просто не се сдържах!
Поздравления!
Върнете се в началото
Вижте профила на потребителя Изпратете лично съобщение
Titu_Andrescu
Напреднал


Регистриран на: 28 Oct 2006
Мнения: 370

Репутация: 68.9
гласове: 29

МнениеПуснато на: Sat Jan 19, 2008 1:57 pm    Заглавие:

Еха, браво..
Върнете се в началото
Вижте профила на потребителя Изпратете лично съобщение
Мирослав Стоенчев
Напреднал


Регистриран на: 21 Aug 2007
Мнения: 279

Репутация: 72
гласове: 45

МнениеПуснато на: Sat Jan 19, 2008 7:44 pm    Заглавие:

Задача 66. Ще докажем, че при положителни [tex]x,y,z[/tex] за които [tex]\min\left\{x,y,z\right\}=x[/tex] е в сила неравенството

[tex]\ \frac{1}{4}-\left\(\frac{yz}{(z+x)(x+y)}-\frac{1}{2}\right\)^{2}+\frac{x(x+y+z)\left\[x^{2}(y+z)+(x+y+z)^{2}(2x+y+z)\right\]}{(x+y)(y+z)(z+x)(2x+y+z)^{2}}-\frac{x^{2}}{2(y+z)^{2}}-\frac{x^{2}(x+y+z)\left\[(y+z)(x+y+z)(2x+y+z)+2x\left\((x+y+z)^{2}-yz\right\)\right\]}{(z+x)^{2}(x+y)^{2}(2x+y+z)^{2}}\ >\ 0.[/tex]

Понеже неравенството е хомогенно, то б.о.о. ще считаме, че [tex]x+y+z=1.[/tex] Ще докажем, че

[tex]i)[/tex] [tex]\frac{1}{4}-\left\(\frac{yz}{(z+x)(x+y)}-\frac{1}{2}\right\)^{2}-\frac{x^{2}}{2(y+z)^{2}}>0[/tex]

[tex]ii)[/tex] [tex]\frac{x(x+y+z)\left\[x^{2}(y+z)+(x+y+z)^{2}(2x+y+z)\right\]}{(x+y)(y+z)(z+x)(2x+y+z)^{2}}-\frac{x^{2}(x+y+z)\left\[(y+z)(x+y+z)(2x+y+z)+2x\left\((x+y+z)^{2}-yz\right\)\right\]}{(z+x)^{2}(x+y)^{2}(2x+y+z)^{2}}\ >\ 0.[/tex]

Полагаме [tex]\ t=yz\Rightarrow x^{2}\le t\le \left\(\frac{1-x}{2}\right\)^{2},\ u=1-x,\ 0<x\le \frac{1}{3},\ \frac{2}{3}\le u<1.[/tex] Получаваме

[tex]\frac{1}{4}-\left\(\frac{yz}{(z+x)(x+y)}-\frac{1}{2}\right\)^{2}-\frac{x^{2}}{2(y+z)^{2}}>0\Leftrightarrow H(x,t)=xt^{2}+2(2x-1)t+x^{3}<0,\ x^{2}\le t\le \left\(\frac{1-x}{2}\right\)^{2}.[/tex]

Имаме [tex]\max_{x^{2}\le t\le \left\(\frac{1-x}{2}\right\)^{2}}H(x,t)=\max_{0\le x\le \frac{1}{3}}\left\{H(x,x^{2}),H(x,\left\(\frac{1-x}{2}\right\)^{2})\right\}=\max_{\frac{2}{3}\le u\le1}\left\{H\left\((1-u),(1-u)^{2}\right\),H(1-u,\frac{u^{2}}{4})\right\}[/tex]

Пресмятаме [tex]H(x,x^{2})=x^{2}(x^{3}+5x-2)<0,\ H(1-u,\frac{u^{2}}{4})=-u^{5}+u^{4}-32u^{3}+56u^{2}-48u+16\le \max_{\frac{2}{3}\le u\le1}(-u^{5}+u^{4}-32u^{3}+56u^{2}-48u+16)<-\frac{1}{2}<0.[/tex]
С това доказахме [tex]i).[/tex]

Сега [tex]\ ii)\Leftrightarrow \frac{1+x+x^{2}-x^{3}}{1-x}>\frac{x\left\(1-x^{2}+2x(1-t)\right\)}{x+t}\Leftrightarrow (1+x+x^{2}-x^{3})(x+t)>x(1-x)\left\(1-x^{2}+2x(1-t)\right\).[/tex]

Но [tex](1+x+x^{2}-x^{3})(x+t)>(1+x+x^{2}-x^{3})x>x(1-x)(1-x^{2}+2x)>x(1-x)\left\(1-x^{2}+2x(1-t)\right\)\Leftrightarrow x^{2}(3-x)>0.[/tex] С това доказахме [tex]ii).[/tex]

Накрая от [tex]i)[/tex] и [tex]ii)[/tex] следва изходното неравенство. Твърдението е доказано.
Върнете се в началото
Вижте профила на потребителя Изпратете лично съобщение
Мирослав Стоенчев
Напреднал


Регистриран на: 21 Aug 2007
Мнения: 279

Репутация: 72
гласове: 45

МнениеПуснато на: Wed Jan 23, 2008 12:27 pm    Заглавие:

Задача 78. Формулировката на задачата я прави нерешима. Да разгледаме триъгълник със страни 3,4,5. За него не е възможно да определим множеството от стойностите на [tex]t\in R[/tex] така,
че да е в сила неравенството от условието.

Предполагам, че 78-ма задача трябва да се формулира така:

Задача 78. Да се опредялят всички стойности на [tex]t\in R,[/tex] за които съществува [tex]\lambda_{t} \in R,[/tex] така, че неравенството [tex]\frac{m_{a}m_{b}m_{c}}{abc}\ge \lambda_{t}\frac{m_{a}^{t}+m_{b}^{t}+m_{c}^{t}}{a+b+c} [/tex] да е в сила за произволен триъгълник.

При тази формулировка задачата има тривиално решение [tex]t\in R,\ \lambda_{t}\le 0. [/tex] Неравенството от условието е еквивалентно на

[tex]\frac{\sqrt{(2a^{2}+2b^2-c^2)(2b^2+2c^2-a^2)(2c^2+2a^2-b^2)}}{abc}\ge \lambda_{t}\frac{(2a^{2}+2b^2-c^2)^{\frac{t}{2}}+(2b^{2}+2c^2-a^2)^{\frac{t}{2}}+(2c^{2}+2a^2-b^2)^{\frac{t}{2}}}{2^{t-3}(a+b+c)}.[/tex]

Сега да фиксираме [tex]t_{0}[/tex] и да определим [tex]\lambda_{t_{0}}.[/tex] За тази цел разглеждаме множеството от триъгълници, за които [tex]a+b+c=1,\ a=b=y\Rightarrow \frac{1}{4}<y<\frac{1}{2}.[/tex]

За тях трябва да бъде изпълнено:[tex]\ \sqrt{4y-1}\ge \lambda_{t_{0}}\frac{y^{2}(1-2y)\left\[(4y-1)^{\frac{t}{2}}+2(9y^{2}-8y+2)^{\frac{t}{2}}\right\]}{2^{t-3}(9y^{2}-8y+2)}[/tex]

Пресмятаме [tex]\lim_{y\rightarrow \frac{1}{4}+0}\sqrt{4y-1}=0,\ \lim_{y\rightarrow \frac{1}{4}+0}\lambda_{t_{0}}\frac{y^{2}(1-2y)\left\[(4y-1)^{\frac{t_{0}}{2}}+2(9y^{2}-8y+2)^{\frac{t_{0}}{2}}\right\]}{2^{t-3}(9y^{2}-8y+2)}= \left\{\begin{array}{ll}\frac{3^{t_{0}-2}}{8^{t_{0}-1}}\lambda_{t_{0}}, & t_{0}>0 \\\frac{4\lambda_{t_{0}}}{3}, & t_{0}=0 \\ sgn(\lambda_{t_{0}})\infty, & t_{0} <0 \end{array}\right. [/tex]

Заключаваме: при произволно [tex]t_{0}\in R\Rightarrow \lambda_{t_{0}}\le 0.[/tex] Окончателно, решения на задачата са двойките [tex](t,\lambda_{t}):\ t\in R,\ \lambda_{t}\le 0.[/tex]
Върнете се в началото
Вижте профила на потребителя Изпратете лично съобщение
Мирослав Стоенчев
Напреднал


Регистриран на: 21 Aug 2007
Мнения: 279

Репутация: 72
гласове: 45

МнениеПуснато на: Wed Jan 23, 2008 7:36 pm    Заглавие:

Задача 50. Трябва да докажем неравенството: [tex]\frac{m_{a}m_{b}}{ab}+\frac{m_{b}m_{c}}{bc}+\frac{m_{c}m_{a}}{ca}\ge \frac{9}{4}.[/tex]

Неравенството е хомогенно, тогава б.о.о. ще считаме, че [tex]a+b+c=1.[/tex]

Неравенството е еквивалентно на [tex]\sum_{cyclic}\frac{\sqrt{(2b^2+2c^2-a^2)(2c^2+2a^2-b^2)}}{ab}\ge 9.[/tex]

Полагаме [tex]G(x,y,z)=\sum_{cyclic}\frac{\sqrt{(2y^2+2z^2-x^2)(2z^2+2x^2-y^2)}}{xy},\ x+y+z=1,\ \frac{1}{3}\le \max\left\{x,y,z\right\}<\frac{1}{2}.[/tex]

[tex]*[/tex]

Нека [tex]x+2y=1,\ \frac{1}{4}<y<\frac{1}{2}.[/tex] Пресмятаме [tex]G(x,y,y)=\frac{2\sqrt{(2x^2+y^2)(4y^2-x^2)}}{xy}+\frac{2x^2+y^2}{y^2}\ge 9\Leftrightarrow (3y-1)^{2}(y-1)^{2}\ge 0.[/tex]

Доказахме, че [tex]G(x,y,y)\ge 9.[/tex]

[tex]**[/tex]

Нека сега [tex]x+y+z=1,\ \min\left\{x,y,z\right\}=x,\ \frac{1}{3}\le \max\left\{x,y,z\right\}<\frac{1}{2}. [/tex] Пресмятаме разликата [tex]G(x,y,z)-G(x,\frac{y+z}{2},\frac{y+z}{2})=k_{1}+k_{2}+k_{3},[/tex] където

[tex]k_{1}=\frac{(y-z)^{2}\left\[k_{z}k_{x}-4z^{2}k_{y+z}+8yzk_{z}\right\]+(y-z)z(4yz-x^2)(8x^{2}+8y^2+z^2+7yz)}{xz(y+z)\left\[(y+z)\sqrt{k_{z}k_{x}}+2z\sqrt{k_{y+z}k_{x,y+z}}\right]}[/tex]

[tex]k_{2}=\frac{(y-z)^{2}\left\[k_{x}k_{y}-4y^{2}k_{y+z}+8yzk_{y}\right\]-(y-z)y(4yz-x^2)(8x^{2}+y^2+8z^2+7yz)}{xy(y+z)\left\[(y+z)\sqrt{k_{x}k_{y}}+2y\sqrt{k_{y+z}k_{x,y+z}}\right\]}[/tex]

[tex]k_{3}=\frac{(y-z)^{2}\left\[(y+z)^{2}+4yz\right]k_{y}k_{z}+16y^{2}z^{2}(k_{y}k_{z}-4x^2)}{yz(y+z)^{2}\left\[(y+z)^{2}\sqrt{k_{y}k_{z}}+8x^{2}yz\right\]}-1[/tex]

Но [tex]k_{1}+k_{2}+k_{3}\ge 0 [/tex] при [tex]\min\left\{x,y,z\right\}=x\Rightarrow G(x,y,z)-G(x,\frac{y+z}{2},\frac{y+z}{2})\ge 0\Leftrightarrow G(x,y,z)\ge G(x,\frac{y+z}{2},\frac{y+z}{2})\ge \min_{\frac{1}{4}<y<\frac{1}{2}}G(x,y,y)=9\Rightarrow G(x,y,z)\ge 9.[/tex] Твърдението е доказано.
Върнете се в началото
Вижте профила на потребителя Изпратете лично съобщение
Мирослав Стоенчев
Напреднал


Регистриран на: 21 Aug 2007
Мнения: 279

Репутация: 72
гласове: 45

МнениеПуснато на: Fri Jan 25, 2008 1:05 am    Заглавие:

Задача 80. Нека [tex]l_{a},l_{b},l_{c},\ r_{a},r_{b},r_{c}[/tex] са съответно дължините на вътрешните ъглополовящи и радиусите на външно-вписаните окръжности за произволен триъгълник.
Да се докаже, че [tex]\frac{r_{a}}{l_{b}+l_{c}}+\frac{r_{b}}{l_{c}+l_{a}}+\frac{r_{c}}{l_{a}+l_{b}}\ge \frac{3}{2}.[/tex]
(Мирослав Стоенчев)
Върнете се в началото
Вижте профила на потребителя Изпратете лично съобщение
Мирослав Стоенчев
Напреднал


Регистриран на: 21 Aug 2007
Мнения: 279

Репутация: 72
гласове: 45

МнениеПуснато на: Fri Feb 22, 2008 1:32 pm    Заглавие:

Задача 81. [tex]ABCD[/tex] е четириъгълник с полупериметър [tex]p,[/tex] който е вписан в окръжност [tex]k(O,R)[/tex] и описан около окръжност [tex]w(I,r).[/tex]

Да се докаже, че [tex]\ p\ge 2r+2\sqrt{r^2+2f(R,r)},[/tex]

където [tex]\ f(R,r)=\sqrt{R^2+r^2-r\sqrt{4R^2+r^2}}\left\(\sqrt{R^2-\frac{r^2\left\(r+\sqrt{4R^2+r^2}\right\)^2}{4R^2}}-\sqrt{R^2+r^2-r\sqrt{4R^2+r^2}}\right\).[/tex]
(Авторска)
Върнете се в началото
Вижте профила на потребителя Изпратете лично съобщение
Мирослав Стоенчев
Напреднал


Регистриран на: 21 Aug 2007
Мнения: 279

Репутация: 72
гласове: 45

МнениеПуснато на: Fri Feb 22, 2008 4:56 pm    Заглавие:

Задача 82. [tex]ABCD[/tex] е четириъгълник с лице [tex]S[/tex] и медицентър [tex]G,[/tex] който е вписан в окръжност [tex]k(O,R)[/tex] и описан около окръжност [tex]w(I,r).[/tex]
Нека [tex]\ |AC|=e,\ |BD|=f;\ \ f(R,r)[/tex] е дефинирана в задача 81.

Да се докаже, че [tex]\ |GI|\le \frac{|e-f|}{2}\sqrt{\frac{f(R,r)}{2S}}[/tex]
Върнете се в началото
Вижте профила на потребителя Изпратете лично съобщение
Мирослав Стоенчев
Напреднал


Регистриран на: 21 Aug 2007
Мнения: 279

Репутация: 72
гласове: 45

МнениеПуснато на: Sun Feb 24, 2008 12:07 pm    Заглавие:

Задача 83. [tex]ABCD[/tex] е четириъгълник с полупериметър [tex]p,[/tex] който е вписан в окръжност [tex]k(O,R)[/tex] и описан около окръжност [tex]w(I,r).[/tex]

Да се докаже, че [tex]\ r+\sqrt{r^2+2f(R,r)}\le \frac{p}{2}\le \sqrt{R^2-r^2+r\sqrt{4R^2+r^2}}.[/tex]
Върнете се в началото
Вижте профила на потребителя Изпратете лично съобщение
Мирослав Стоенчев
Напреднал


Регистриран на: 21 Aug 2007
Мнения: 279

Репутация: 72
гласове: 45

МнениеПуснато на: Sat Mar 01, 2008 11:39 am    Заглавие:

Задача 84. [tex]ABCD[/tex] е четириъгълник с полупериметър [tex]p,[/tex] който е вписан в окръжност [tex]k(O,R)[/tex] и описан около окръжност [tex]w(I,r).[/tex] Нека [tex]|AC|=e,\ |BD|=f.[/tex]

Да се докаже, че [tex]0\le 2p^2-(e+f)^2\le 4R^2-\frac{r^2}{R^2}\left\(r+\sqrt{4R^2+r^2}\right\)^2.[/tex]
Върнете се в началото
Вижте профила на потребителя Изпратете лично съобщение
Пафнутий
VIP


Регистриран на: 04 Mar 2008
Мнения: 1199

Репутация: 137.7
гласове: 54

МнениеПуснато на: Thu Mar 13, 2008 10:02 pm    Заглавие:

Не знам дали тази задача е за тука, но просто самото решение е уникално WinkПоне на мен ми се струва такова Embarassed

Задача 85 Да се докаже, ако a >0, b > 0, c>0, то

[tex] ab^5 + bc^5 +ca^5\ge abc(a^2b +b^2c + c^2a)[/tex]

(Унгарско списание по математика за средношколци) Laughing
Върнете се в началото
Вижте профила на потребителя Изпратете лично съобщение
Мирослав Стоенчев
Напреднал


Регистриран на: 21 Aug 2007
Мнения: 279

Репутация: 72
гласове: 45

МнениеПуснато на: Sat Mar 15, 2008 8:32 pm    Заглавие:

Относно задача 85 - Иван Тонов написа цяла статия за "унгарската задача", брой 2 на
списание Математика от 2005г. Задачата може да бъде решена
и по други начини, но сега
съм на тема Комбинаторна Геометрия, авторски задачи Very Happy


Задача 86. За всяко естествено число [tex]N,[/tex] означаваме с [tex]k(N) [/tex]минималния брой "дами",
които могат да бъдат поставени на шахматна дъска
с размери [tex]N\times N,[/tex] така че:
[tex]i)[/tex] никои две дами не се атакуват
[tex]ii)[/tex] всяко поле на дъската се атакува от поне една дама.

Да се докаже, че [tex]\ \lim_{N\to\infty}\ \frac{k(N)}{N}=\frac{2}{3}.[/tex]
Върнете се в началото
Вижте профила на потребителя Изпратете лично съобщение
krassi_holmz
Редовен


Регистриран на: 05 Jan 2006
Мнения: 146
Местожителство: Ню Йорк, BG
Репутация: 57.9
гласове: 18

МнениеПуснато на: Thu Apr 03, 2008 3:46 am    Заглавие:

Ето и от мене един звяр:
Задача 87: За положителни числа,
[tex]3 (a b+c b+a c)^2+7 \left(a^4+b^4+c^4\right)\geq 8 \left((b+c) a^3+ (a+c)b^3+(a+b) c^3\right)[/tex]
[EDIT] Преди имаше грешка, сега я оправих.
Върнете се в началото
Вижте профила на потребителя Изпратете лично съобщение
Мирослав Стоенчев
Напреднал


Регистриран на: 21 Aug 2007
Мнения: 279

Репутация: 72
гласове: 45

МнениеПуснато на: Mon Apr 07, 2008 9:09 pm    Заглавие:

Задача 87 (Краси)
Д-во: Б.о.о. [tex]\ a+b+c=1,\ \min\left\{a,b,c\right\}=a,\ t=bc,\ u=1-a\Rightarrow 0<t\le \left\(\frac{b+c}{2}\right\)^2=\left\(\frac{1-a}{2}\right\)^2=\frac{u^2}{4}.[/tex]

Пресмятаме: [tex]b^2+c^2=u^2-2t,\ b^3+c^3=u^3-3ut,\ b^4+c^4=u^4-4u^2t+2t^2.[/tex]

Изходното неравенство е еквивалентно на: [tex]G(t)=33t^2-(66u^2-30u)t+33u^4-66u^3+69u^2-36u+7\ge0,\ 0<t\le \frac{u^2}{4},\ \frac{2}{3}\le u<1.[/tex]

[tex]G'(t)=0\Leftrightarrow t=\frac{66u^2-30u}{66}=\frac{11u^2-5u}{11}>\frac{u^2}{4}\Rightarrow \min_{0<t\le \frac{u^2}{4}}G(t)=G(\frac{u^2}{4})=(3u-2)^2\left\(33u^2-60u+28\right\)\ge 0.[/tex]
Равенство се достига само при [tex]u=\frac{2}{3}\Rightarrow a=b=c.[/tex]
Върнете се в началото
Вижте профила на потребителя Изпратете лично съобщение
Titu_Andrescu
Напреднал


Регистриран на: 28 Oct 2006
Мнения: 370

Репутация: 68.9
гласове: 29

МнениеПуснато на: Tue Nov 04, 2008 5:15 pm    Заглавие:

Задача 85. Решение. [tex]a,b,c>0[/tex].
Ср.Аритметично-Ср.Геометрично:

[tex]\frac{\underbrace{ab^5+..+ab^5}_{8}+\underbrace{bc^5+bc^5}_{2}+\underbrace{ca^5+..+ca^5}_{11}}{21}\ge \sqrt[21]{a^{63}b^{42}c^{21}}=a^3b^2c[/tex](1).

Аналогично [tex]8bc^5+2ca^5+11ab^5\ge ab^3c^2[/tex](2) и [tex]8ca^5+2ab^5+11bc^5\ge a^2bc^3[/tex](3).

Събираме почленно резултати (1), (2),(3) и получаваме, че [tex]\frac{21ab^5+21bc^5+21ca^5}{21}\ge a^3b^2c+ab^3c^2+a^2bc^3=abc(a^2b+b^2c+c^2a)[/tex],
което искахме да докажем.
Върнете се в началото
Вижте профила на потребителя Изпратете лично съобщение
Пафнутий
VIP


Регистриран на: 04 Mar 2008
Мнения: 1199

Репутация: 137.7
гласове: 54

МнениеПуснато на: Mon Aug 17, 2009 4:53 pm    Заглавие:

Titu_Andrescu написа:

[tex]25\sum_{cyclic}x^6+230\sum_{cyclic}x^5y+115\sum_{cyclic}x^4y^2+10\sum_{cyclic}x^3y^3+80\sum_{cyclic}x^4yz\ge 336\sum_{cyclic}x^3y^2z+124\sum_{cyclic}x^2y^2z^2[/tex].

Последното неравенство вече е вярно от Теоремата на Muirhead! С това твърдението е доказано.
Не исках да се връщам към толкова стари постове, но теоремата на Мюрхед е в сила за симетрични суми, а не за циклични.
Върнете се в началото
Вижте профила на потребителя Изпратете лично съобщение
martosss
VIP Gold


Регистриран на: 17 Mar 2007
Мнения: 3937
Местожителство: Somewhere over the rainbow
Репутация: 424.2Репутация: 424.2
гласове: 213

МнениеПуснато на: Sat Sep 05, 2009 3:14 pm    Заглавие:

Titu_Andrescu написа:

Задача 18. Нека [tex]a,b,c[/tex] са половителни реални числа. Да се докаве, че [tex]\frac{(2a+b+c)^2}{2a^2+(b+c)^2}+\frac{(2b+c+a)^2}{2b^2+(c+a)^2}+\frac{(2c+a+b)^2}{2c^2+(a+b)^2}\le 8[/tex].
(Национална олимпиада по математика САЩ 2003г.)

Нека от двете страни извадим 9:

[tex]\frac{(2a+b+c)^2}{2a^2+(b+c)^2}-3+\frac{(2b+c+a)^2}{2b^2+(c+a)^2}-3+\frac{(2c+a+b)^2}{2c^2+(a+b)^2}-3\le -1[/tex]
Привеждаме 3-ките под общ знаменател и получаваме:
[tex]\sum_{cyc}\; \frac{\left[4a^2+4a(b+c)+(b+c)^2\right]-3\left[2a^2+(b+c)^2\right]}{2a^2+(b+c)^2}\le -1[/tex]
[tex]\sum_{cyc}\; -2\frac{a^2-2a(b+c)+(b+c)^2}{2a^2+(b+c)^2}\le-1[/tex]
[tex]\sum_{cyc}\; \frac{(a-(b+c))^2}{2a^2+(b+c)^2}\ge \frac{1}{2}[/tex]
Сега понеже
[tex]2bc\le b^2+c^2\Right[/tex]
[tex]2a^2+(b+c)^2\le 2(a^2+b^2+c^2)[/tex]
Аналогично постъпваме с другите два знаменателя, с което остава да докажем, че:
[tex]\frac{(a-b-c)^2+(b-a-c)^2+(c-a-b)^2}{\N 2(a^2+b^2+c^2)}\ge \frac{1}{\N 2}[/tex]
Това е еквивалентно на
[tex](a-b)^2+(b-c)^2+(c-a)^2\ge 0[/tex].
С това задачата е решена.

Забележка. При това решение неравенството е в сила за всички [tex]a,b,c\in R[/tex], освен за (0,0,0),(a,-a,0),(0,b,-b),(a,0,-a),
Върнете се в началото
Вижте профила на потребителя Изпратете лично съобщение
martosss
VIP Gold


Регистриран на: 17 Mar 2007
Мнения: 3937
Местожителство: Somewhere over the rainbow
Репутация: 424.2Репутация: 424.2
гласове: 213

МнениеПуснато на: Sat Sep 05, 2009 4:14 pm    Заглавие:

Titu_Andrescu написа:
Задача 19. Ако [tex]a,b,c[/tex] са положителни реални числа и [tex]a+b+c=1[/tex], то да се докаже, че [tex]a\sqrt[3]{1+b-c}+b\sqrt[3]{1+c-a}+c\sqrt[3]{1+a-b}\le 1[/tex].
(Национална олимпиада по математика Япония 2005г. Финали)

От неравенството на Хьолдер за [tex]\frac{2}{3}+\frac{1}{3}=1[/tex]имаме
[tex]a^{\frac{2}{3}}\sqrt[3]{a+ab-ac}+b^{\frac{2}{3}}\sqrt[3]{b+bc-ab}+c^{\frac{2}{3}}\sqrt[3]{c+ac-bc}\le \left(a^{{\frac{2}{3}\frac{3}{2}}}+b^{{\frac{2}{3}\frac{3}{2}}}+c^{ {\frac{2}{3}\frac{3}{2}}}\right)^{\frac{2}{3}}\left(\underbrace{a+b+c}_{1}+ {ab+bc+ac-ac-ab-bc}\right)^{\frac{1}{3}}=1[/tex]
Върнете се в началото
Вижте профила на потребителя Изпратете лично съобщение
martosss
VIP Gold


Регистриран на: 17 Mar 2007
Мнения: 3937
Местожителство: Somewhere over the rainbow
Репутация: 424.2Репутация: 424.2
гласове: 213

МнениеПуснато на: Tue Sep 08, 2009 2:04 pm    Заглавие:

Titu_Andrescu написа:
Задача 39. Да се докаже неравенството на Carlson. Нека [tex]a,b,c[/tex] са неотрицателни реални числа. Да се докаже, че
[tex]\sqrt[3]{\frac{(a+b)(b+c)(c+a)}{8}}\ge \sqrt{\frac{ab+bc+ca}{3}}[/tex].

След повдигане на 6-та степен и умножаване по 64*27 получаваме:
[tex]27(2abc+ab(a+b)+bc(b+c)+ac(a+c))^2\ge 64(ab+bc+ac)^3\\27((a+b+c)(ab+bc+ac)-abc)^2\ge 64(ab+bc+ac)^3[/tex]
Нека положим [tex]a+b+c=p\\ab+bc+ac=q\\abc=r[/tex]
[tex]27(pq-r)^2\ge 64q^3\;\; (1)[/tex]
От СА-СГ имаме
[tex]p\ge 3r^{\frac{1}{3}\;\; (2)[/tex]
От Средно Аритметично-Средно Хармонично имаме
[tex]q\ge 3r^{\frac{2}{3}}\;\; (3)[/tex]
От (2) и (3) следва, че
[tex]pq\ge 9r\Right r\le \frac{pq}{9}[/tex]
Тогава за (1) получаваме
[tex]LHS\ge 27(\frac{8}{9}pq)^2=27\frac{64p^2q^2}{81}=\frac{64p^2q^2}{3}\;\; (4)[/tex]
Сега понеже [tex](a+b+c)^2\ge 3(ab+bc+ac)\;\;\;\; (p^2\ge 3q)[/tex] e eквивалентно на [tex](a-b)^2+(b-c)^2+(a-c)^2\ge 0[/tex], то за (4) получаваме

[tex]\frac{64}{3}p^2q^2\ge \frac{64}{3}*3q^3=64q^3[/tex].
С това доказахме (1), откъдето следва исканото неравенство.
Върнете се в началото
Вижте профила на потребителя Изпратете лично съобщение
Titu_Andrescu
Напреднал


Регистриран на: 28 Oct 2006
Мнения: 370

Репутация: 68.9
гласове: 29

МнениеПуснато на: Sun Sep 13, 2009 3:42 pm    Заглавие:

Задача 86. Да се докаже неравенството [tex]x+sinx\ge 2log(1+x)[/tex], за [tex]x>-1[/tex].
Предложена от Kee-Wai Lau, Hong Kong.
Задачата е много е лесна.
Върнете се в началото
Вижте профила на потребителя Изпратете лично съобщение
Покажи мнения от преди:   
   Форум за математика Форуми -> Олимпиади и състезания за 9-12 клас Часовете са според зоната GMT + 2 Часа
Иди на страница Предишна  1, 2, 3, 4, 5, 6, 7, 8  Следваща
Страница 7 от 8

 
Идете на:  
Не Можете да пускате нови теми
Не Можете да отговаряте на темите
Не Можете да променяте съобщенията си
Не Можете да изтривате съобщенията си
Не Можете да гласувате в анкети
Може да прикачвате файлове
Може да сваляте файлове от този форум
Copyright © 2005-2021 math10.com.